Optimisation using constraints

Click For Summary
The discussion focuses on using Lagrange multipliers to find the highest and lowest points on the intersection of an elliptic paraboloid and a right circular cylinder. The initial setup leads to a value of λ = 4, which raises concerns about the elimination of Lagrange multipliers. Participants emphasize the need to consider cases where y equals zero and to ensure all necessary equations are included in the calculations. Additionally, an alternative approach is suggested, involving direct substitution of the constraint to simplify the problem. The conversation highlights the importance of careful algebraic manipulation and thorough consideration of all cases in optimization problems.
lagwagon555
Messages
59
Reaction score
1

Homework Statement



Consider the intersection of two surfaces: an elliptic paraboloid
z = x2 + 2x + 4y2 and a right circular cylinder x2 + y2 = 1. Use Lagrange multipliers to find
the highest and lowest points on the curve of intersection

The Attempt at a Solution



L = x^2 + 2x + 4y^2 - λ(x^2 + y^2 - 1)
Lx = 2x + 2 - 2λx = 0
Ly = 8y - 2λy = 0

Rearranging gives the bizarre result of λ = 4. Aren't I supposed to be able to eliminate the lagrange multipliers by it giving me λ in terms of y? It's giving me a value instead.

I ran with this anyway, and got the answer of x = 1/3, y = +-0.94, but I don't think this is correct. I suspect I'm doing something wrong algebraically? Any help hugely appreciated :)
 
Physics news on Phys.org
lagwagon555 said:
I suspect I'm doing something wrong algebraically? Any help hugely appreciated :)

You need to be careful with the last equation. I believe you divided by y, but that's valid only if y isn't zero. You need to explore both possibilities. By the way, you forgot an equation. What about Lλ?

What makes you think you have the wrong answer?
 
Last edited:
If you don't trust the method try it directly as well. Use the constraint to eliminate y^2 in z. Now look at the extrema for x in [-1,1]. You do need to consider the y=0 case as well.
 
Question: A clock's minute hand has length 4 and its hour hand has length 3. What is the distance between the tips at the moment when it is increasing most rapidly?(Putnam Exam Question) Answer: Making assumption that both the hands moves at constant angular velocities, the answer is ## \sqrt{7} .## But don't you think this assumption is somewhat doubtful and wrong?

Similar threads

  • · Replies 2 ·
Replies
2
Views
2K
  • · Replies 9 ·
Replies
9
Views
2K
Replies
2
Views
2K
  • · Replies 5 ·
Replies
5
Views
2K
Replies
5
Views
2K
  • · Replies 8 ·
Replies
8
Views
2K
  • · Replies 8 ·
Replies
8
Views
3K
  • · Replies 6 ·
Replies
6
Views
2K
  • · Replies 10 ·
Replies
10
Views
2K
  • · Replies 16 ·
Replies
16
Views
3K